You are on page 1of 32

The

Ateneo Law
Entrance
Exam 2017
A Guide to Preparation
I WANT TO TAKE THE EXAM! WHAT ARE THE REQUIREMENTS?
----------------------------------------------------------------------------------------------------
Applicant must have obtained a Bachelor’s Degree in Arts or Sciences, or a higher academic
degree, from an authorized and recognized university or college.
The applicant must have earned at least eighteen (18) units of English, six (6) units of
Mathematics, eighteen (18) units of Social Science subjects, and at least three (3) units of a
Rizal.
The applicant must accomplish the online application form, at www.law.ateneo.edu and
take the entrance examination.
He or she should pass the school’s entrance examination. For admission to the
examination, the following must be submitted:
Photocopy of collegiate transcript or records showing compliance with the eligibility
requirements stated above.
If graduation is not yet reflected in the transcript, a true copy of grades or a
certification of candidacy for graduation must be appended to the collegiate
transcript.
Three (3) I.D. pictures, 2”x 2” in size with red background.
Payment of testing fee of Three Thousand Pesos (Php3,000.00).
Two (2) recommendations coming from one academic recommendation from 1) the
dean or a faculty member of his or her school and 2) one recommendation from a
reputable source
The applicant must submit all requirements on or before February 10, 2017 at the Office
of the Dean, 3/F Ateneo Professional Schools Building, 20 Rockwell Drive, Rockwell
Center, Makati City, Philippines. Office hours are from Monday to Friday, 1:00 p.m. – 7:00
p.m.
The applicant will be assigned his/her entrance exam schedule upon submission by the
applicant of all admission requirements.
The applicant must take the Philippine Law School Admission Test (PhilSAT). Please
check with the Legal Education Board for more details.
NOW THAT I KNOW THE REQUIREMENTS, ANY TIPS ON PREPARING
FOR ANSWERING THE TEST?
----------------------------------------------------------------------------------------------------

Answer as many sample exams as you can. This will not only give you an idea of what to
expect in the real exam but it will also develop your speed and accuracy in answering the
test!
RELAX, DON’T PANIC! Most applicants fail the exam because they panic too much and lose
their focus. Just breathe and you’ll be able to answer more efficiently!
There will be NO law questions. The exam will test your logic, deductive and reasoning
skills!
Get enough rest the night before the exam and make sure you have a full stomach so that
you can concentrate well!
You might be surprised but most of the questions are answerable by common sense. As
long as you remember and know how to apply what you learned in English, you’ll be fine!
Last year there wasn’t a Math portion in the exam, but it wouldn’t hurt to practice just in
case!
In preparation, it would be useful to practice your reading by taking online speed reading
tests or reading books. It would definitely work to your advantage when answering the
Reading Comprehension portion of the exam!
For the Logic part of the exam, try drawing figures, symbols or letters on a separate sheet
of paper so that it will be easier for you to visualize the problem, instead of being
overwhelmed with the facts. (Example: If the problem involves 3 groups, you can try
drawing 3 circles)!
Questions are in multiple choice format, so if push comes to shove and you really don’t
know the answer, use deductive reasoning!
Be conscious of the time. If you really can’t answer a problem, mark it and move on to the
next!
WHAT IS REGINA IUSTITIAE?
----------------------------------------------------------------------------------------------------

Regina Iustitiae Sorority is all about women empowerment. We are an independent, non-profit
organization of women from the Ateneo School of Law with the vision of producing excellent
socially-oriented women lawyers who will make a genuine difference in Philippine society.

Translated to mean “Queen of Justice,” Regina Iustitiae is an organization of women who are
strong leaders, devoted to upholding justice and to excelling in their respective fields.

We are proud to to be the oldest sorority in the Ateneo School of Law, having been founded in
2005 and in such a short time, we have been able to produce 2 bar topnotchers (Majesty Eve
Jala placed 4th while April Love Regis placed 7th in the 2008 Bar Examinations) and around 65
lawyers now working in top law firms and government offices in the country. This can be
attributed to the sorority’s excellent Bar Operations Program, which is all expense-paid as well
as its Academic Assistance Program, providing reviewers, mock exams and tutorials for resident
sisses.

Aside from this, the sorority is also actively engaged in socially-oriented activities not only within
the law school but also outside, as evidenced by its outreach programs, such as reaching out to
women in conflict with the law, visiting cancer patients and bringing joy to kids in Philippine
General Hospital through “Walls of Hope.”

The sorority has also organized several successful fundraising events, including its big
anniversary parties and “Verdict”, a tie-up with UP Med School’s Phi Kappa Mu Fraternity as well
as other sororities and fraternities.

Finally, Regina Iustitiae believes in well-roundedness and bringing out the best in each member.
We work together and inspire each other to succeed. True enough, a lot of Regina sisses have
made it to the Dean’s List, are leaders in their own organizations (be it the Student Council,
Ateneo Human Rights Center, Ateneo Society of International Law, The Palladium, among
others), debaters and mooters, and internationally recognized awardees. This only proves that
even in the law school, it is indeed possible to strike a healthy balance between academics and
extra-curricular activities as the sorority has always endeavored to do.

For more information about Regina Iustitiae, you may visit www.reginaiustitiaesorority.com.
4. Any of the following pairs could be the third and
fourth digits, respectively, of an acceptable product
SECTION I code, EXCEPT:
Time—35 minutes (A) 0, 1
23 Questions (B) 0, 3
(C) 1, 0
Directions: Each group of questions in this section is (D) 3, 0
based on a set of conditions. In answering some of the (E) 3, 4
questions, it may be useful to draw a rough diagram.
Choose the best answer. 5. Which one of the following must be true about any
acceptable product code?
Questions 1–5 (A) There is exactly one digit between the digit 0 and
A company employee generates a series of five-digit the digit 1.
product codes in accordance with the following rules: (B) There is exactly one digit between the digit 1 and
The codes use the digits 0, 1, 2, 3, and 4, and no others. the digit 2.
Each digit occurs exactly once in any code. (C) There are at most two digits between the digit 1
The second digit has a value exactly twice that of the and the digit 3.
first digit. (D) There are at most two digits between the digit 2
The value of the third digit is less than the value of the and the digit 3.
fifth digit. (E) There are at most two digits between the digit 2
and the digit 4.
1. If the last digit of an acceptable product code is 1, it
must be true that the Questions 6–10
(A) first digit is 2 Exactly three films—Greed, Harvest, and Limelight—
(B) second digit is 0 are shown during a film club’s festival held on
(C) third digit is 3 Thursday, Friday, and Saturday. Each film is shown at
(D) fourth digit is 4 least once during the festival but never more than
(E) fourth digit is 0 once on a given day. On each day at least one film is
shown. Films are shown one at a time. The following
2. Which one of the following must be true about any conditions apply:
acceptable product code? On Thursday Harvest is shown, and no film is shown
(A) The digit 1 appears in some position before the after it on that day.
digit 2. On Friday either Greed or Limelight, but not both, is
(B) The digit 1 appears in some position before the shown, and no film is shown after it on that day.
digit 3. On Saturday either Greed or Harvest, but not both, is
(C) The digit 2 appears in some position before the shown, and no film is shown after it on that day.
digit 3.
(D) The digit 3 appears in some position before the 6. Which one of the following could be a complete and
digit 0. accurate description of the order in which the films are
(E) The digit 4 appears in some position before the shown at the festival?
digit 3. (A) Thursday: Limelight, then Harvest; Friday:
Limelight; Saturday: Harvest
3. If the third digit of an acceptable product code is not (B) Thursday: Harvest; Friday: Greed, then Limelight;
0, which one of the following must be true? Saturday: Limelight, then Greed
(A) The second digit of the product code is 2. (C) Thursday: Harvest; Friday: Limelight; Saturday:
(B) The third digit of the product code is 3. Limelight, then Greed
(C) The fourth digit of the product code is 0. (D) Thursday: Greed, then Harvest, then Limelight;
(D) The fifth digit of the product code is 3. Friday: Limelight; Saturday: Greed
(E) The fifth digit of the product code is 1. (E) Thursday: Greed, then Harvest; Friday: Limelight,
then Harvest; Saturday: Harvest
Jamaica will not be its destination in week 4.
7. Which one of the following CANNOT be true? Trinidad will be its destination in week 7.
(A) Harvest is the last film shown on each day of the Freedom will make exactly two voyages to Martinique,
festival. and at least one voyage to Guadeloupe will occur in
(B) Limelight is shown on each day of the festival. some week between those two voyages.
(C) Greed is shown second on each day of the festival. Guadeloupe will be its destination in the week
(D) A different film is shown first on each day of the preceding any voyage it makes to Jamaica.
festival. No destination will be scheduled for consecutive
(E) A different film is shown last on each day of the weeks.
festival.
11. Which one of the following is an acceptable
8. If Limelight is never shown again during the festival schedule of destinations for Freedom, in order from
once Greed is shown, then which one of the following week 1 through week 7?
is the maximum number of film showings that could (A) Guadeloupe, Jamaica, Martinique, Trinidad,
occur during the festival? Guadeloupe, Martinique, Trinidad
(A) three (B) Guadeloupe, Martinique, Trinidad, Martinique,
(B) four Guadeloupe, Jamaica, Trinidad
(C) five (C) Jamaica, Martinique, Guadeloupe, Martinique,
(D) six Guadeloupe, Jamaica, Trinidad
(E) seven (D) Martinique, Trinidad, Guadeloupe, Jamaica,
Martinique, Guadeloupe, Trinidad
9. If Greed is shown exactly three times, Harvest is (E) Martinique, Trinidad, Guadeloupe, Trinidad,
shown exactly twice, and Limelight is shown exactly Guadeloupe, Jamaica, Martinique
once, then which one of the following must be true?
(A) All three films are shown on Thursday. 12. Which one of the following CANNOT be true about
(B) Exactly two films are shown on Saturday. Freedom’s schedule of voyages?
(C) Limelight and Harvest are both shown on (A) Freedom makes a voyage to Trinidad in week 6.
Thursday. (B) Freedom makes a voyage to Martinique in week 5.
(D) Greed is the only film shown on Saturday. (C) Freedom makes a voyage to Jamaica in week 6.
(E) Harvest and Greed are both shown on Friday. (D) Freedom makes a voyage to Jamaica in week 3.
(E) Freedom makes a voyage to Guadeloupe in week 3.
10. If Limelight is shown exactly three times, Harvest
is shown exactly twice, and Greed is shown exactly 13. If Freedom makes a voyage to Trinidad in week 5,
once, then which one of the following is a complete which one of the following could be true?
and accurate list of the films that could be the first film (A) Freedom makes a voyage to Trinidad in week 1.
shown on Thursday? (B) Freedom makes a voyage to Martinique in week 2.
(A) Harvest (C) Freedom makes a voyage to Guadeloupe in week
(B) Limelight 3
(C) Greed, Harvest (D) Freedom makes a voyage to Martinique in week 4.
(D) Greed, Limelight (E) Freedom makes a voyage to Jamaica in week 6.
(E) Greed, Harvest, Limelight
14. If Freedom makes a voyage to Guadeloupe in week
Questions 11–17 1, and a voyage to Jamaica in week 5, which one of the
A cruise line is scheduling seven week-long voyages following must be true?
for the ship Freedom. Each voyage will occur in exactly (A) Freedom makes a voyage to Jamaica in week 2.
one of the first seven weeks of the season: weeks 1 (B) Freedom makes a voyage to Trinidad in week 2.
through 7. Each voyage will be to exactly one of four (C) Freedom makes a voyage to Martinique in week 3.
destinations: Guadeloupe, Jamaica, Martinique, or (D) Freedom makes a voyage to Guadeloupe in week 6
Trinidad. Each destination will be scheduled for at (E) Freedom makes a voyage to Martinique in week 6.
least one of the weeks. The following conditions apply
to Freedom’s schedule:
15. If Freedom makes a voyage to Guadeloupe in week (B) Center 1: glass, newsprint, tin; Center 2: glass,
1 and to Trinidad in week 2, which one of the following newsprint, tin; Center 3: newsprint, plastic, wood
must be true? (C) Center 1: glass, newsprint, wood; Center 2: glass,
(A) Freedom makes a voyage to Martinique in week 3. newsprint, tin; Center 3: plastic, tin
(B) Freedom makes a voyage to Martinique in week 4. (D) Center 1: glass, plastic, tin; Center 2: glass, tin;
(C) Freedom makes a voyage to Martinique in week 5. Center 3: newsprint, wood
(D) Freedom makes a voyage to Guadeloupe in week 3 (E) Center 1: newsprint, plastic, wood; Center 2:
(E) Freedom makes a voyage to Guadeloupe in week 5. newsprint, plastic, wood; Center 3: glass,
newsprint, tin
16. If Freedom makes a voyage to Martinique in week
3, which one of the following could be an accurate list 19. Which one of the following is a complete and
of Freedom’s destinations in week 4 and week 5, accurate list of the recycling centers in Rivertown any
respectively? one of which could recycle plastic?
(A) Guadeloupe, Trinidad (A) Center 1 only
(B) Jamaica, Guadeloupe (B) Center 3 only
(C) Martinique, Trinidad (C) Center 1, Center 2
(D) Trinidad, Jamaica (D) Center 1, Center 3
(E) Trinidad, Martinique (E) Center 1, Center 2, Center 3

17. Which one of the following must be true about 20. If Center 2 recycles three kinds of material, then
Freedom’s schedule of voyages? which one of the following kinds of material must
(A) Freedom makes a voyage to Guadeloupe either in Center 3
week 1 or else in week 2. recycle?
(B) Freedom makes a voyage to Martinique either in (A) glass
week 2 or else in week 3. (B) newsprint
(C) Freedom makes at most two voyages to (C) plastic
Guadeloupe. (D) tin
(D) Freedom makes at most two voyages to Jamaica. (E) wood
(E) Freedom makes at most two voyages to Trinidad.
21. If each recycling center in Rivertown recycles
exactly three kinds of material, then which one of the
following could be true?
Questions 18–23 (A) Only Center 2 recycles glass.
There are exactly three recycling centers in Rivertown: (B) Only Center 3 recycles newsprint.
Center 1, Center 2, and Center 3. Exactly five kinds of (C) Only Center 1 recycles plastic.
material are recycled at these recycling centers: glass, (D) Only Center 3 recycles tin.
newsprint, plastic, tin, and wood. Each recycling center (E) Only Center 1 recycles wood.
recycles at least two but no more than three of these
kinds of material. The following conditions must hold: 22. If Center 3 recycles glass, then which one of the
Any recycling center that recycles wood also recycles following kinds of material must Center 2 recycle?
newsprint. Every kind of material that Center 2 (A) glass
recycles is also recycled at Center 1. Only one of the (B) newsprint
recycling centers recycles plastic, and that recycling (C) plastic
center does not recycle glass. (D) tin
(E) wood
18. Which one of the following could be an accurate
account of all the kinds of material recycled at each 23. If Center 1 is the only recycling center that recycles
recycling center in Rivertown? wood, then which one of the following could be a
(A) Center 1: newsprint, plastic, wood; Center 2: complete and accurate list of the kinds of material that
newsprint, wood; Center 3: glass, tin, wood
one of the recycling centers recycles? (A) plastic, tin
(B) newsprint, wood STOP
(C) newsprint, tin IF YOU FINISH BEFORE TIME IS CALLED, YOU MAY
(D) glass, wood CHECK YOUR WORK ON THIS SECTION ONLY.
(E) glass, tin DO NOT WORK ON ANY OTHER SECTION IN THE TEST.
SECTION II as the sense of security of the retained employees.
Time—35 minutes Which one of the following most accurately expresses
25 Questions the main conclusion of the economist’s argument?

Directions: The questions in this section are based on (A) If an action taken to secure the survival of a
the reasoning contained in brief statements or business fails to enhance the welfare of the
passages. For some questions, more than one of the business’s employees, that action cannot be good for
choices could conceivably answer the question. the business as a whole.
However, you are to choose the best answer; that (B) Some measures taken by a business to increase
is, the response that most accurately and completely productivity fail to be beneficial to the business as a
answers the question. You should not make whole.
assumptions that are by commonsense standards (C) Only if the employees of a business are also its
implausible, superfluous, or incompatible with the owners will the interests of the employees and owners
passage. Choose the best answer. coincide, enabling measures that will be beneficial to
the business as a whole.
1. Economist: Every business strives to increase its (D) There is no business that does not make efforts to
productivity, for this increases profits for the owners increase its productivity.
and the likelihood that the business will survive. But (E) Decreasing the number of employees in a business
not all efforts to increase productivity are beneficial to undermines the sense of security of retained
the business as a whole. Often, attempts to increase employees.
productivity decrease the number of employees,
which clearly harms the dismissed employees as well 2. All Labrador retrievers bark a great deal. All Saint
Bernards bark infrequently. Each of Rosa’s dogs is a
cross between a Labrador retriever and a Saint them are very badly made. Therefore, half of the
Bernard. Therefore, Rosa’s dogs are moderate dresses in this closet are Kenisha’s and half of them
barkers. Which one of the following uses flawed are Connie’s.
reasoning that most closely resembles the flawed
reasoning used in the argument above? 3. A century in certain ways is like a life, and as the end
(A) All students who study diligently make good of a century approaches, people behave toward that
grades. But some students who do not study diligently century much as someone who is nearing the end of
also make good grades. Jane studies somewhat life does toward that life. So just as people in their last
diligently. Therefore, Jane makes somewhat good years spend much time looking back on the events of
grades. their life, people at a century’s end _______.
(B) All type A chemicals are extremely toxic to human Which one of the following most logically completes
beings. All type B chemicals are nontoxic to human the argument?
beings. This household cleaner is a mixture of a type (A) reminisce about their own lives
A chemical and a type B chemical. Therefore, this (B) fear that their own lives are about to end
household cleaner is moderately toxic. (C) focus on what the next century will bring
(C) All students at Hanson School live in Green County. (D) become very interested in the history of the
All students at Edwards School live in Winn County. century just ending
Members of the Perry family attend both Hanson and (E) reflect on how certain unfortunate events of the
Edwards. Therefore, some members of the Perry century could have been avoided
family live in Green County and some live in Winn
County. 4. Consumer: The latest Connorly Report suggests
(D) All transcriptionists know shorthand. All engineers that Ocksenfrey prepackaged meals are virtually
know calculus. Bob has worked both as a devoid of nutritional value. But the Connorly Report is
transcriptionist and as an engineer. Therefore, Bob commissioned by Danto Foods, Ocksenfrey’s largest
knows both shorthand and calculus. corporate rival, and early drafts of the report are
(E) All of Kenisha’s dresses are very well made. All of submitted for approval to Danto Foods’ public
Connie’s dresses are very badly made. Half of the relations department. Because of the obvious bias of
dresses in this closet are very well made, and half of this report, it is clear that Ocksenfrey’s prepackaged
meals really are nutritious. The reasoning in the (A) Anyone with a master’s degree and without a
consumer’s argument is most vulnerable to criticism felony conviction is eligible for appointment to
on the grounds that the argument the executive board.
(A) treats evidence that there is an apparent bias as (B) Only candidates eligible for appointment to the
evidence that the Connorly Report’s claims are false executive board can be accepted for the position of
(B) draws a conclusion based solely on an Executive Administrator.
unrepresentative sample of Ocksenfrey’s products (C) An undergraduate degree is not necessary for
(C) fails to take into account the possibility that acceptance for the position of Executive
Ocksenfrey has just as much motivation to create Administrator.
negative publicity for Danto as Danto has to create (D) If Murray did not have a felony conviction, he would
negative publicity for Ocksenfrey be accepted for the position of Executive
(D) fails to provide evidence that Danto Foods’ Administrator.
prepackaged meals are not more nutritious than (E) The felony charge on which Murray was convicted
Ocksenfrey’s are is relevant to the duties of the position of Executive
(E) presumes, without providing justification, that Administrator.
Danto Foods’ public relations department would not
approve a draft of a report that was hostile to Danto 7. Ethicist: The most advanced kind of moral
Foods’ products motivation is based solely on abstract principles. This
form of motivation is in contrast with calculated self
5. Scientist: Earth’s average annual temperature has interest or the desire to adhere to societal norms and
increased by about 0.5 degrees Celsius over the last conventions. The actions of which one of the following
century. This warming is primarily the result of the individuals exhibit the most advanced kind of moral
buildup of minor gases in the atmosphere, blocking motivation, as described by the ethicist?
the outward flow of heat from the planet. Which one of (A) Bobby contributed money to a local charity during
the following, if true, would count as evidence against a charity drive at work because he worried that not
the scientist’s explanation of Earth’s warming? doing so would make him look stingy.
(B) Wes contributed money to a local charity during a
(A) Only some of the minor gases whose presence charity drive at work because he believed that doing
in the atmosphere allegedly resulted in the so would improve his employer’s opinion of him.
phenomenon described by the scientist were (C) Donna’s employers engaged in an illegal but
produced by industrial pollution. profitable practice that caused serious damage to the
(B) Most of the warming occurred before 1940, while environment. Donna did not report this practice to the
most of the buildup of minor gases in the atmosphere authorities, out of fear that her employers would
occurred after 1940. retaliate against her.
(C) Over the last century, Earth received slightly more (D) Jadine’s employers engaged in an illegal but
solar radiation in certain years than it did in others. profitable practice that caused serious damage to the
(D) Volcanic dust and other particles in the environment. Jadine reported this practice to the
atmosphere reflect much of the Sun’s radiation back authorities out of a belief that protecting the
into space before it can reach Earth’s surface. environment is always more important than monetary
(E) The accumulation of minor gases in the profit.
atmosphere has been greater over the last century (E) Leigh’s employers engaged in an illegal but
than at any other time in Earth’s history. profitable practice that caused serious damage to the
environment. Leigh reported this practice to the
6. An undergraduate degree is necessary for authorities only because several colleagues had been
appointment to the executive board. Further, no one pressuring her to do so.
with a felony conviction can be appointed to the board.
Thus, Murray, an accountant with both a bachelor’s 8. Proponents of the electric car maintain that when
and a master’s degree, cannot be accepted for the the technical problems associated with its battery
position of Executive Administrator, since he has a design are solved, such cars will be widely used and,
felony conviction. The argument’s conclusion follows because they are emission-free, will result in an
logically if which one of the following is assumed?
abatement of the environmental degradation caused hold.
by auto emissions. But unless we dam more rivers, the (B) It is advisable for scientists to use double-blind
electricity to charge these batteries will come from techniques in as high a proportion of their experiments
nuclear or coal-fired power plants. Each of these three as they can.
power sources produces considerable environmental (C) Scientists sometimes neglect to adequately
damage. Thus, the electric car _______. consider the risk of misinterpreting evidence on the
Which one of the following most logically completes basis of prior expectations and opinions.
the argument? (D) Whenever possible, scientists should refrain from
(A) will have worse environmental consequences than interpreting evidence on the basis of previously
its proponents may believe formed expectations and convictions.
(B) will probably remain less popular than other types (E) Double-blind experimental techniques are often
of cars an effective way of ensuring scientific objectivity.
(C) requires that purely technical problems be solved
before it can succeed 11. It is now a common complaint that the electronic
(D) will increase the total level of emissions rather media have corroded the intellectual skills required
than reduce it and fostered by the literary media. But several
(E) will not produce a net reduction in environmental centuries ago the complaint was that certain
degradation intellectual skills, such as the powerful memory and
extemporaneous eloquence that were intrinsic to oral
9. Although video game sales have increased steadily culture, were being destroyed by the spread of literacy.
over the past 3 years, we can expect a reversal of this So, what awaits us is probably a mere alteration of the
trend in the very near future. Historically, over three human mind rather than its devolution. The reference
quarters of video games sold have been purchased by to the complaint of several centuries ago that powerful
people from 13 to 16 years of age, and the number of memory and extemporaneous eloquence were being
people in this age group is expected to decline steadily destroyed plays which one of the following
over the next 10 years. roles in the argument?
Which one of the following, if true, would most (A) evidence supporting the claim that the
seriously weaken the argument? intellectual skills fostered by the literary media
(A) Most people 17 years old or older have never are being destroyed by the electronic media
purchased a video game. (B) an illustration of the general hypothesis being
(B) Video game rentals have declined over the past advanced that intellectual abilities are
3 years. inseparable from the means by which people
(C) New technology will undoubtedly make entirely communicate
new entertainment options available over the next 10 (C) an example of a cultural change that did not
years. necessarily have a detrimental effect on the
(D) The number of different types of video games human mind overall
available is unlikely to decrease in the near future. (D) evidence that the claim that the intellectual
(E) Most of the people who have purchased video skills required and fostered by the literary
games over the past 3 years are over the age of 16. media are being lost is unwarranted
(E) possible evidence, mentioned and then dismissed,
10. Double-blind techniques should be used whenever that might be cited by supporters of the hypothesis
possible in scientific experiments. They help prevent being criticized
the misinterpretations that often arise due to
expectations and opinions that scientists already 12. Suppose I have promised to keep a confidence and
hold, and clearly scientists should be extremely someone asks me a question that I cannot answer
diligent in trying to avoid such misinterpretations. truthfully without thereby breaking the promise.
Which one of the following most accurately expresses Obviously, I cannot both keep and break the same
the main conclusion of the argument? promise. Therefore, one cannot be obliged both to
(A) Scientists’ objectivity may be impeded by answer all questions truthfully and to keep all
interpreting experimental evidence on the basis promises.
of expectations and opinions that they already Which one of the following arguments is most similar
in its reasoning to the argument above? cans as L. The conclusion of the argument follows
(A) It is claimed that we have the unencumbered logically if which one of the following is assumed?
right to say whatever we want. It is also (A) The aluminum in the cans of M cannot be
claimed that we have the obligation to be civil recycled further.
to others. But civility requires that we not (B) Recycled aluminum is of poorer quality than
always say what we want. So, it cannot be true unrecycled aluminum.
both that we have the unencumbered right to (C) All of the aluminum in an aluminum can is
say whatever we want and that we have the recovered when the can is recycled.
duty to be civil. (D) None of the soft-drink cans in group L had been
(B) Some politicians could attain popularity with made from recycled aluminum.
voters only by making extravagant promises; (E) Aluminum soft-drink cans are more easily
this, however, would deceive the people. So, recycled than are soft-drink cans made from
since the only way for some politicians to be other materials.
popular is to deceive, and any politician needs
to be popular, it follows that some politicians 14. A cup of raw milk, after being heated in a
must deceive. microwave oven to 50 degrees Celsius, contains half
(C) If we put a lot of effort into making this report its initial concentration of a particular enzyme,
look good, the client might think we did so lysozyme. If, however, the milk reaches that
because we believed our proposal would not temperature through exposure to a conventional heat
stand on its own merits. On the other hand, if source of 50 degrees Celsius, it will contain nearly all
we do not try to make the report look good, of its initial concentration of the enzyme. Therefore,
the client might think we are not serious about what destroys the enzyme is not heat but microwaves,
her business. So, whatever we do, we risk her which generate heat.
criticism. Which one of the following, if true, most seriously
(D) If creditors have legitimate claims against a weakens the argument?
business and the business has the resources to (A) Heating raw milk in a microwave oven to a
pay those debts, then the business is obliged to temperature of 100 degrees Celsius destroys
pay them. Also, if a business has obligations to nearly all of the lysozyme initially present in
pay debts, then a court will force it to pay that milk.
them. But the courts did not force this business (B) Enzymes in raw milk that are destroyed through
to pay its debts, so either the creditors did not excessive heating can be replaced by adding
have legitimate claims or the business did not enzymes that have been extracted from other
have sufficient resources. sources.
(E) If we extend our business hours, we will either (C) A liquid exposed to a conventional heat source
have to hire new employees or have existing of exactly 50 degrees Celsius will reach that
employees work overtime. But both new temperature more slowly than it would if it
employees and additional overtime would were exposed to a conventional heat source
dramatically increase our labor costs. We hotter than 50 degrees Celsius.
cannot afford to increase labor costs, so we (D) Milk that has been heated in a microwave oven
will have to keep our business hours as they does not taste noticeably different from milk
stand. that has been briefly heated by exposure to a
conventional heat source.
13. Standard aluminum soft-drink cans do not vary in (E) Heating any liquid by microwave creates small
the amount of aluminum that they contain. Fifty zones within it that are much hotter than the
percent of the aluminum contained in a certain group overall temperature that the liquid will
(M) of standard aluminum soft-drink cans was ultimately reach.
recycled from another group (L) of used, standard
aluminum softdrink cans. Since all the cans in L were 15. A new government policy has been developed to
recycled into cans in M and since the amount of avoid many serious cases of influenza. This goal will
material other than aluminum in an aluminum can is be accomplished by the annual vaccination of high-
negligible, it follows that M contains twice as many risk individuals: everyone 65 and older as well as
anyone with a chronic disease that might cause them results that are not inherently suspect.
to experience complications from the influenza virus. (E) If inherently suspect claims are usually false,
Each year’s vaccination will protect only against the then the majority of claims made by scientists
strain of the influenza virus deemed most likely to be are false as well.
prevalent that year, so every year it will be necessary
for all high-risk individuals to receive a vaccine for a 17. Hospital executive: At a recent conference on
different strain of the virus. nonprofit
Which one of the following is an assumption that management, several computer experts
would allow the conclusion above to be properly maintained that the most significant threat faced
drawn? by large institutions such as universities and
(A) The number of individuals in the high-risk hospitals is unauthorized access to confidential
group for influenza will not significantly data. In light of this testimony, we should make
change from year to year. the protection of our clients’ confidentiality our
(B) The likelihood that a serious influenza epidemic highest priority.
will occur varies from year to year. The hospital executive’s argument is most vulnerable
(C) No vaccine for the influenza virus protects to
against more than one strain of that virus. which one of the following objections?
(D) Each year the strain of influenza virus deemed (A) The argument confuses the causes of a problem
most likely to be prevalent will be one that had with the appropriate solutions to that problem.
not previously been deemed most likely to be (B) The argument relies on the testimony of experts
prevalent. whose expertise is not shown to be sufficiently
(E) Each year’s vaccine will have fewer side effects broad to support their general claim.
than the vaccine of the previous year since the (C) The argument assumes that a correlation
technology for making vaccines will constantly between two phenomena is evidence that one is
improve. the cause of the other.
(D) The argument draws a general conclusion about
16. Taylor: Researchers at a local university claim that a group based on data about an
61 percent of the information transferred during a unrepresentative sample of that group.
conversation is communicated through nonverbal (E) The argument infers that a property belonging to
signals. But this claim, like all such large institutions belongs to all institutions.
mathematically precise claims, is suspect, because
claims of such exactitude could never be 18. Modern science is built on the process of posing
established by science. hypotheses and testing them against observations—
Sandra: While precision is unobtainable in many areas in essence, attempting to show that the hypotheses
of life, it is commonplace in others. Many are incorrect. Nothing brings more recognition than
scientific disciplines obtain extremely precise overthrowing conventional wisdom. It is accordingly
results, which should not be doubted merely unsurprising that some scientists are skeptical of the
because of their precision. widely accepted predictions of global warming. What
The statements above provide the most support for is instead remarkable is that with hundreds of
holding that Sandra would disagree with Taylor about researchers striving to make breakthroughs in
which one of the following statements? climatology, very few find evidence that global
(A) Research might reveal that 61 percent of the warming is unlikely.
information taken in during a conversation is The information above provides the most support for
communicated through nonverbal signals. which one of the following statements?
(B) It is possible to determine whether 61 percent of (A) Most scientists who are reluctant to accept the
the information taken in during a conversation global warming hypothesis are not acting in
is communicated through nonverbal signals. accordance with the accepted standards of
(C) The study of verbal and nonverbal scientific debate.
communication is an area where one cannot (B) Most researchers in climatology have substantial
expect great precision in one’s research results. motive to find evidence that would discredit
(D) Some sciences can yield mathematically precise the global warming hypothesis.
(C) There is evidence that conclusively shows that (B) questioning a claim supported by statistical data
the global warming hypothesis is true. by arguing that statistical data can be manipulated to
(D) Scientists who are skeptical about global support whatever view the interpreter wants to
warming have not offered any alternative support
hypotheses to explain climatological data. (C) attempting to refute an argument by showing that,
(E) Research in global warming is primarily driven contrary to what has been claimed, the truth of the
by a desire for recognition in the scientific premises does not guarantee the truth of the
community. conclusion
(D) criticizing a view on the grounds that the view
19. Historian: The Land Party achieved its only is based on evidence that is in principle impossible to
national victory in Banestria in 1935. It received most disconfirm
of its support that year in rural and semirural areas, (E) attempting to cast doubt on a conclusion by
where the bulk of Banestria’s population lived at the claiming that the statistical sample on which the
time. The economic woes of the years surrounding conclusion is based is too small to be dependable
that election hit agricultural and small business
interests the hardest, and the Land Party specifically 21. Driver: My friends say I will one day have an
targeted those groups in 1935. I conclude that the accident because I drive my sports car recklessly. But
success of the Land Party that year was due to the I have done some research, and apparently minivans
combination of the Land Party’s specifically and larger sedans have very low accident rates
addressing the concerns of these groups and the compared to sports cars. So trading my sports car in
depth of the economic problems people in these for a minivan would lower my risk of having an
groups were facing. Each of the following, if true, accident. The reasoning in the driver’s argument is
strengthens the historian’s argument EXCEPT: most vulnerable to criticism on the grounds that this
(A) In preceding elections the Land Party made no argument
attempt to address the interests of economically (A) infers a cause from a mere correlation
distressed urban groups. (B) relies on a sample that is too narrow
(B) Voters are more likely to vote for a political (C) misinterprets evidence that a result is likely as
party that focuses on their problems. evidence that the result is certain
(C) The Land Party had most of its successes when (D) mistakes a condition sufficient for bringing about
there was economic distress in the agricultural sector. a result for a condition necessary for doing so
(D) No other major party in Banestria specifically (E) relies on a source that is probably not well-
addressed the issues of people who lived in informed
semirural areas in 1935.
(E) The greater the degree of economic distress 22. Editorialist: News media rarely cover local politics
someone is in, the more likely that person is to vote. thoroughly, and local political business is usually
conducted secretively. These factors each tend to
20. Gamba: Muñoz claims that the Southwest isolate local politicians from their electorates. This
Hopeville Neighbors Association overwhelmingly has the effect of reducing the chance that any
opposes the new water system, citing this as evidence particular act of resident participation will elicit a
of citywide opposition. The association did pass a positive official response, which in turn discourages
resolution opposing the new water system, but only 25 resident participation in local politics. Which one of
of 350 members voted, with 10 in favor of the system. the following is most strongly supported by the
Furthermore, the 15 opposing votes represent far less editorialist’s statements?
than 1 percent of Hopeville’s population. One should (A) Particular acts of resident participation would be
not assume that so few votes represent the view of the likely to elicit a positive response from local politicians
majority of Hopeville’s residents. Of the following, if those politicians were less isolated from their
which one most accurately describes Gamba’s electorate.
strategy of argumentation? (B) Local political business should be conducted less
(A) questioning a conclusion based on the results of secretively because this would avoid discouraging
a vote, on the grounds that people with certain views resident participation in local politics.
are more likely to vote
(C) The most important factor influencing a resident’s (E) Consumer input affects external rather than
decision as to whether to participate in local politics is internal design components of cars.
the chance that the participation will elicit a positive
official response. 25. During the nineteenth century, the French
(D) More-frequent thorough coverage of local politics academy of art was a major financial sponsor of
would reduce at least one source of discouragement painting and sculpture in France; sponsorship by
from resident participation in local politics. private individuals had decreased dramatically by this
(E) If resident participation in local politics were not time. Because the academy discouraged innovation in
discouraged, this would cause local politicians to be the arts, there was little innovation in nineteenth
less isolated from their electorate. century French sculpture. Yet nineteenth century
French painting showed a remarkable degree of
23. Philosopher: An action is morally right if it would innovation.
be reasonably expected to increase the aggregate Which one of the following, if true, most helps to
well-being of the people affected by it. An action is explain the difference between the amount of
morally wrong if and only if it would be reasonably innovation in French painting and the amount of
expected to reduce the aggregate wellbeing of the innovation in French sculpture during the nineteenth
people affected by it. Thus, actions that would be century?
reasonably expected to leave unchanged the (A) In France in the nineteenth century, the French
aggregate well-being of the people affected by them academy gave more of its financial support to painting
are also right. The philosopher’s conclusion follows than it did to sculpture.
logically if which one of the following is assumed? (B) The French academy in the nineteenth century
(A) Only wrong actions would be reasonably financially supported a greater number of sculptors
expected to reduce the aggregate well-being of than painters, but individual painters received more
the people affected by them. support, on average, than individual sculptors.
(B) No action is both right and wrong. (C) Because stone was so much more expensive than
(C) Any action that is not morally wrong is morally paint and canvas, far more unsponsored paintings
right. were produced than were unsponsored sculptures in
(D) There are actions that would be reasonably France during the nineteenth century.
expected to leave unchanged the aggregate (D) Very few of the artists in France in the nineteenth
well-being of the people affected by them. century who produced sculptures also produced
(E) Only right actions have good consequences. paintings.
(E) Although the academy was the primary sponsor of
24. Car companies solicit consumer information on sculpture and painting, the total amount of financial
such human factors as whether a seat is comfortable support that French sculptors and painters received
or whether a set of controls is easy to use. However, from sponsors declined during the nineteenth century.
designer interaction with consumers is superior to STOP
survey data; the data may tell the designer why a IF YOU FINISH BEFORE TIME IS CALLED, YOU MAY
feature on last year’s model was given a low rating, but CHECK YOUR WORK ON THIS SECTION ONLY.
data will not explain how that feature needs to be DO NOT WORK ON ANY OTHER SECTION IN THE TEST.
changed in order to receive a higher rating. The
reasoning above conforms most closely to which
one of the following propositions?
(A) Getting consumer input for design modifications
can contribute to successful product design.
(B) Car companies traditionally conduct extensive
postmarket surveys.
(C) Designers aim to create features that will appeal to
specific market niches.
(D) A car will have unappealing features if consumers
are not consulted during its design stage.
SECTION III (B) Shortly after the new water heater was installed,
Time—35 minutes Jimmy’s uncle came to live with him, doubling the size
25 Questions of the household.
(C) After having done his laundry at a laundromat,
Directions: The questions in this section are based on Jimmy bought and started using a gas dryer when he
the reasoning contained in brief statements or replaced his water heater.
passages. For some questions, more than one of the (D) Jimmy’s utility company raised the rates for gas
choices could conceivably answer the question. consumption following installation of the new water
However, you are to choose the best answer; that is, heater.
the response that most accurately and completely (E) Unusually cold weather following installation of the
answers the question. You should not make new water heater resulted in heavy gas
assumptions that are by commonsense standards usage.
implausible, superfluous, or incompatible with the
passage. Choose the best answer 3. Carolyn: The artist Marc Quinn has displayed,
behind a glass plate, biologically replicated fragments
1. Situation: Someone living in a cold climate buys a of Sir John Sulston’s DNA, calling it a “conceptual
winter coat that is stylish but not warm in order to portrait” of Sulston. But to be a portrait, something
appear sophisticated. must bear a recognizable resemblance to its subject.
Analysis: People are sometimes willing to sacrifice Arnold: I disagree. Quinn’s conceptual portrait is a
sensual comfort or pleasure for the sake of maximally realistic portrait, for it holds actual
appearances. instructions according to which Sulston was created.
The analysis provided for the situation above is most The dialogue provides most support for the claim that
appropriate for which one of the following situations? Carolyn and Arnold disagree over whether the object
(A) A person buys an automobile to commute to work described by Quinn as a conceptual portrait of Sir
even though public transportation is quick and John Sulston
reliable. (A) should be considered to be art
(B) A parent buys a car seat for a young child because (B) should be considered to be Quinn’s work
it is more colorful and more comfortable for the child (C) bears a recognizable resemblance to Sulston
than the other car seats on the market, though no (D) contains instructions according to which Sulston
safer. was created
(C) A couple buys a particular wine even though their (E) is actually a portrait of Sulston
favorite wine is less expensive and better tasting
because they think it will impress their dinner guests. 4. Many corporations have begun decorating their
(D) A person sets her thermostat at a low temperature halls with motivational posters in hopes of boosting
during the winter because she is concerned about the their employees’ motivation to work productively.
environmental damage caused by using fossil fuels to However, almost all employees at these corporations
heat her home. are already motivated to work productively. So these
(E) An acrobat convinces the circus that employs him corporations’ use of motivational posters is unlikely to
to purchase an expensive outfit for him so that he can achieve its intended purpose.
wear it during his act to impress the audience. The reasoning in the argument is most vulnerable to
criticism on the grounds that the argument
2. After replacing his old gas water heater with a new, (A) fails to consider whether corporations that do not
pilotless, gas water heater that is rated as highly currently use motivational posters would increase
efficient, Jimmy’s gas bills increased. their employees’ motivation to work productively if
Each of the following, if true, contributes to an they began using the posters
explanation of the increase mentioned above EXCEPT: (B) takes for granted that, with respect to their
(A) The new water heater uses a smaller percentage of employees’ motivation to work productively,
the gas used by Jimmy’s household than did the old corporations that decorate their halls with
one. motivational posters are representative of
corporations in general
(C) fails to consider that even if motivational posters (E) An action must have broad community support
do not have one particular beneficial effect for if it is to be successful.
corporations, they may have similar effects that are
equally beneficial 7. Antonio: One can live a life of moderation by never
(D) does not adequately address the possibility that deviating from the middle course. But then one loses
employee productivity is strongly affected by factors the joy of spontaneity and misses the opportunities
other than employees’ motivation to work productively that come to those who are occasionally willing to
(E) fails to consider that even if employees are already take great chances, or to go too far.
motivated to work productively, motivational posters Marla: But one who, in the interests of moderation,
may increase that motivation never risks going too far is actually failing to live a life
of moderation: one must be moderate even in one’s
5. Atrens: An early entomologist observed ants moderation.
carrying particles to neighboring ant colonies and Antonio and Marla disagree over
inferred that the ants were bringing food to their (A) whether it is desirable for people occasionally
neighbors. Further research, however, revealed that to take great chances in life
the ants were emptying their own colony’s dumping (B) what a life of moderation requires of a person
site. Thus, the early entomologist was wrong. Atrens’s (C) whether it is possible for a person to embrace
conclusion follows logically if which one of the other virtues along with moderation
following is assumed? (D) how often a person ought to deviate from the
(A) Ant societies do not interact in all the same middle course in life
ways that human societies interact. (E) whether it is desirable for people to be
(B) There is only weak evidence for the view that moderately spontaneous
ants have the capacity to make use of objects
as gifts. 8. Advertisement: Fabric-Soft leaves clothes soft and
(C) Ant dumping sites do not contain particles that fluffy, and its fresh scent is a delight. We conducted a
could be used as food. test using over 100 consumers to prove Fabric-Soft is
(D) The ants to whom the particles were brought best. Each consumer was given one towel washed
never carried the particles into their own with Fabric-Soft and one towel washed without it.
colonies. Ninety-nine percent of the consumers preferred the
(E) The entomologist cited retracted his conclusion Fabric-Soft towel. So Fabric-Soft is the most effective
when it was determined that the particles the fabric softener available.
ants carried came from their dumping site. The advertisement’s reasoning is most vulnerable to
criticism on the grounds that it fails to consider
6. Jablonski, who owns a car dealership, has donated whether
cars to driver education programs at area schools for (A) any of the consumers tested are allergic to
over five years. She found the statistics on car fabric softeners
accidents to be disturbing, and she wanted to do (B) Fabric-Soft is more or less harmful to the
something to encourage better driving in young environment than other fabric softeners
drivers. Some members of the community have shown (C) Fabric-Soft is much cheaper or more expensive
their support for this action by purchasing cars from than other fabric softeners
Jablonski’s dealership. (D) the consumers tested find the benefits of using
Which one of the following propositions is best fabric softeners worth the expense
illustrated by the passage? (E) the consumers tested had the opportunity to
(A) The only way to reduce traffic accidents is evaluate fabric softeners other than Fabric-Soft
through driver education programs.
(B) Altruistic actions sometimes have positive 9. Naturalist: The recent claims that the Tasmanian
consequences for those who perform them. tiger is not extinct are false. The Tasmanian tiger’s
(C) Young drivers are the group most likely to natural habitat was taken over by sheep farming
benefit from driver education programs. decades ago, resulting in the animal’s systematic
(D) It is usually in one’s best interest to perform elimination from the area. Since then naturalists
actions that benefit others. working in the region have discovered no hard
evidence of its survival, such as carcasses or tracks. (B) the amount of mercury in a saltwater fish
In spite of alleged sightings of the animal, the depends on the amount of pollution in the
Tasmanian tiger no longer exists. Which one of the ocean habitat of the fish
following is an assumption on which (C) mercury derived from fish is essential for the
the naturalist’s argument depends? normal growth of a seabird’s feathers
(A) Sheep farming drove the last Tasmanian tigers to (D) the stuffed seabirds whose feathers were tested
starvation by chasing them from their natural habitat. for mercury were not fully grown
(B) Some scavengers in Tasmania are capable of (E) the process used to preserve birds in the 1880s
destroying tiger carcasses without a trace. did not substantially decrease the amount of
(C) Every naturalist working in the Tasmanian mercury in the birds’ feathers
tiger’s natural habitat has looked systematically
for evidence of the tiger’s survival. 12. Novel X and Novel Y are both
(D) The Tasmanian tiger did not move and adapt to semiautobiographical novels and contain many very
a different region in response to the loss of habitat. similar themes and situations, which might lead one
(E) Those who have reported sightings of the to suspect plagiarism on the part of one of the
Tasmanian tiger are not experienced naturalists. authors. However, it is more likely that the similarity of
themes and situations in the two novels is merely
10. Advertisers have learned that people are more coincidental, since both authors are from very similar
easily encouraged to develop positive attitudes about backgrounds and have led similar lives. Which one of
thingstoward which they originally have neutral or the following most accurately expresses the
evennegative attitudes if those things are linked, with conclusion drawn in the argument?
pictorial help rather than exclusively through prose, to (A) Novel X and Novel Y are both
things about which they already have positive semiautobiographical novels, and the two novels
attitudes.Therefore, advertisers are likely to _______. contain many very similar themes and situations.
Which one of the following most logically completes (B) The fact that Novel X and Novel Y are both
the argument? semiautobiographical novels and contain many very
(A) use little if any written prose in their similar themes and situations might lead one to
advertisements suspect plagiarism on the part of one of the authors.
(B) try to encourage people to develop positive (C) The author of Novel X and the author of Novel Y are
attitudes about products that can be better from very similar backgrounds and have led very
represented pictorially than in prose similar lives.
(C) place their advertisements on television rather (D) It is less likely that one of the authors of Novel X or
than in magazines Novel Y is guilty of plagiarism than that the similarity
(D) highlight the desirable features of the advertised of themes and situations in the two novels is merely
product by contrasting them pictorially with coincidental.
undesirable features of a competing product (E) If the authors of Novel X and Novel Y are from very
(E) create advertisements containing pictures of similar backgrounds and have led similar lives,
things most members of the target audience like suspicions that either of the authors plagiarized are
very likely to be unwarranted.
11. Feathers recently taken from seabirds stuffed and
preserved in the 1880s have been found to contain 13. Therapist: Cognitive psychotherapy focuses on
only half as much mercury as feathers recently taken changing a patient’s conscious beliefs. Thus,
from living birds of the same species. Since mercury cognitive psychotherapy is likely to be more effective
that accumulates in a seabird’s feathers as the at helping patients overcome psychological problems
feathers grow is derived from fish eaten by the bird, than are forms of psychotherapy that focus on
these results indicate that mercury levels in saltwater changing unconscious beliefs and desires, since only
fish are higher now than they were 100 years ago. conscious beliefs are under the patient’s direct
The argument depends on assuming that conscious control.
(A) the proportion of a seabird’s diet consisting of Which one of the following, if true, would most
fish was not as high, on average, in the 1880s strengthen the therapist’s argument?
as it is today
(A) Psychological problems are frequently caused by (E) A university should not pursue any activity that
unconscious beliefs that could be changed with the would block the achievement of the goals of academic
aid of psychotherapy. scholarship at that university.
(B) It is difficult for any form of psychotherapy to be
effective without focusing on mental states that are 15. A consumer magazine surveyed people who had
under the patient’s direct conscious control. sought a psychologist’s help with a personal problem.
(C) Cognitive psychotherapy is the only form of Of those responding who had received treatment for 6
psychotherapy that focuses primarily on changing the months or less, 20 percent claimed that treatment
patient’s conscious beliefs. “made things a lot better.” Of those responding who
(D) No form of psychotherapy that focuses on had received longer treatment, 36 percent claimed
changing the patient’s unconscious beliefs and that treatment “made things a lot better.” Therefore,
desires can be effective unless it also helps change psychological treatment lasting more than 6 months
beliefs that are under the patient’s direct conscious is more effective than shorter-term treatment. Which
control. one of the following, if true, most seriously weakens
(E) All of a patient’s conscious beliefs are under the the argument?
patient’s conscious control, but other psychological (A) Of the respondents who had received treatment
states cannot be controlled effectively without the aid for longer than 6 months, 10 percent said that
of psychotherapy. treatment made things worse.
(B) Patients who had received treatment for longer
14. Commentator: In academic scholarship, sources than 6 months were more likely to respond to
are always cited, and methodology and theoretical the survey than were those who had received
assumptions are set out, so as to allow critical study, treatment for a shorter time.
replication, and expansion of scholarship. In open- (C) Patients who feel they are doing well in
source software, the code in which the program is treatment tend to remain in treatment, while
written can be viewed and modified by individual users those who are doing poorly tend to quit earlier.
for their purposes without getting permission from the (D) Patients who were dissatisfied with their
producer or paying a fee. In contrast, the code of treatment were more likely to feel a need to
proprietary software is kept secret, and modifications express their feelings about it and thus to
can be made only by the producer, for a fee. This return the survey.
shows that open-source software better matches the (E) Many psychologists encourage their patients to
values embodied in academic scholarship, and since receive treatment for longer than 6 months.
scholarship is central to the mission of universities,
universities should use only open-source software. 16. Philosopher: Nations are not literally persons; they
The commentator’s reasoning most closely conforms have no thoughts or feelings, and, literally speaking,
to which one of the following principles? they perform no actions. Thus they have no moral
(A) Whatever software tools are most advanced and rights or responsibilities. But no nation can survive
can achieve the goals of academic scholarship are the unless many of its citizens attribute such rights and
ones that should alone be used in universities. responsibilities to it, for nothing else could prompt
(B) Universities should use the type of software people to make the sacrifices national citizenship
technology that is least expensive, as long as that type demands. Obviously, then, a nation _______. Which one
of software technology is adequate for the purposes of the following most logically completes
of academic scholarship. the philosopher’s argument?
(C) Universities should choose the type of software (A) cannot continue to exist unless something other
technology that best matches the values embodied in than the false belief that the nation has moral
the activities that are central to the mission of rights motivates its citizens to make sacrifices
universities. (B) cannot survive unless many of its citizens have
(D) The form of software technology that best some beliefs that are literally false
matches the values embodied in the activities that are (C) can never be a target of moral praise or blame
central to the mission of universities is the form of (D) is not worth the sacrifices that its citizens make
software technology that is most efficient for on its behalf
universities to use. (E) should always be thought of in metaphorical
rather than literal terms the example are not mistaken about their
child’s innocence
17. When exercising the muscles in one’s back, it is
important, in order to maintain a healthy back, to 19. Editor: Many candidates say that if elected they
exercise the muscles on opposite sides of the spine will reduce governmental intrusion into voters’ lives.
equally. After all, balanced muscle development is But voters actually elect politicians who instead
needed to maintain a healthy back, since the muscles promise that the government will provide assistance
on opposite sides of the spine must pull equally in to solve their most pressing problems. Governmental
opposing directions to keep the back in proper assistance, however, costs money, and money can
alignment and protect the spine. Which one of the come only from taxes, which can be considered a form
following is an assumption required of governmental intrusion. Thus, governmental
by the argument? intrusion into the lives of voters will rarely be
(A) Muscles on opposite sides of the spine that are substantially reduced over time in a democracy.
equally well developed will be enough to keep Which one of the following, if true, would most
the back in proper alignment. strengthen the editor’s argument?
(B) Exercising the muscles on opposite sides of the (A) Politicians who win their elections usually keep
spine unequally tends to lead to unbalanced their campaign promises.
muscle development. (B) Politicians never promise what they really
(C) Provided that one exercises the muscles on intend to do once in office.
opposite sides of the spine equally, one will (C) The most common problems people have are
have a generally healthy back. financial problems.
(D) If the muscles on opposite sides of the spine are (D) Governmental intrusion into the lives of voters
exercised unequally, one’s back will be is no more burdensome in nondemocratic
irreparably damaged. countries than it is in democracies.
(E) One should exercise daily to ensure that the (E) Politicians who promise to do what they
muscles on opposite sides of the spine keep actually believe ought to be done are rarely
the back in proper alignment. elected.

18. Editorialist: In all cultures, it is almost universally 20. We should accept the proposal to demolish the old
accepted that one has a moral duty to prevent train station, because the local historical society,
members of one’s family from being harmed. Thus, which vehemently opposes this, is dominated by
few would deny that if a person is known by the people who have no commitment to long-term
person’s parents to be falsely accused of a crime, it economic well-being. Preserving old buildings creates
would be morally right for the parents to hide the an impediment to new development, which is critical
accused from the police. Hence, it is also likely to be to economic health. The flawed reasoning exhibited by
widely accepted that it is sometimes morally right to the argument above is most similar to that exhibited
obstruct the police in their work. The reasoning in the by which one of the following arguments?
editorialist’s argument is most vulnerable to criticism (A) Our country should attempt to safeguard works
on the grounds that this argument of art that it deems to possess national cultural
(A) utilizes a single type of example for the purpose significance. These works might not be
of justifying a broad generalization recognized as such by all taxpayers, or even
(B) fails to consider the possibility that other moral all critics. Nevertheless, our country ought to
principles would be widely recognized as expend whatever money is needed to procure
overriding any obligation to protect a family all such works as they become available.
member from harm (B) Documents of importance to local heritage
(C) presumes, without providing justification, that should be properly preserved and archived for
allowing the police to arrest an innocent the sake of future generations. For, if even one
person assists rather than obstructs justice of these documents is damaged or lost, the
(D) takes for granted that there is no moral integrity of the historical record as a whole
obligation to obey the law will be damaged.
(E) takes for granted that the parents mentioned in (C) You should have your hair cut no more than
once a month. After all, beauticians suggest Moreover, the Coffee Shoppe can avoid a decrease in
that their customers have their hair cut twice a overall profitability only if its coffee sales do not
month, and they do this as a way of generating decrease. Which one of the following statements
more business for themselves. follows logically from the statements above?
(D) The committee should endorse the plan to (A) If the Coffee Shoppe’s overall profitability
postpone construction of the new expressway. decreases, the price it pays for coffee beans
Many residents of the neighborhoods that will have continued to increase.
would be affected are fervently opposed to that (B) If the Coffee Shoppe’s overall profitability
construction, and the committee is obligated to decreases, either it will have begun selling
avoid alienating those residents. noncoffee products or its coffee sales will have
(E) One should not borrow even small amounts of decreased.
money unless it is absolutely necessary. Once (C) The Coffee Shoppe’s overall profitability will
one borrows a few dollars, the interest starts to decrease if the price it pays for coffee beans
accumulate. The longer one takes to repay, the continues to increase.
more one ends up owing, and eventually a (D) The price it pays for coffee beans cannot
small debt has become a large one. decrease without the Coffee Shoppe’s overall
profitability also decreasing.
21. Ethicist: On average, animals raised on grain must (E) Either the price it pays for coffee beans will
be fed sixteen pounds of grain to produce one pound continue to increase or the Coffee Shoppe’s
of meat. A pound of meat is more nutritious for coffee sales will increase.
humans than a pound of grain, but sixteen pounds of
grain could feed many more people than could a 23. Political candidates’ speeches are loaded with
pound of meat. With grain yields leveling off, large promises and with expressions of good intention, but
areas of farmland going out of production each year, one must not forget that the politicians’ purpose in
and the population rapidly expanding, we must accept giving these speeches is to get themselves elected.
the fact that consumption of meat will soon be morally Clearly, then, these speeches are selfishly motivated
unacceptable. Which one of the following, if true, and the promises made in them are unreliable. Which
would most weaken the ethicist’s argument? one of the following most accurately describes a flaw
(A) Even though it has been established that a in the argument above?
vegetarian diet can be healthy, many people (A) The argument presumes, without providing
prefer to eat meat and are willing to pay for it. justification, that if a person’s promise is not
(B) Often, cattle or sheep can be raised to maturity selfishly motivated then that promise is
on grass from pastureland that is unsuitable for reliable.
any other kind of farming. (B) The argument presumes, without providing
(C) If a grain diet is supplemented with protein justification, that promises made for selfish
derived from non-animal sources, it can have reasons are never kept.
nutritional value equivalent to that of a diet (C) The argument confuses the effect of an action
containing meat. with its cause.
(D) Although prime farmland near metropolitan (D) The argument overlooks the fact that a promise
areas is being lost rapidly to suburban need not be unreliable just because the person
development, we could reverse this trend by who made it had an ulterior motive for doing so.
choosing to live in areas that are already urban. (E) The argument overlooks the fact that a
(E) Nutritionists agree that a diet composed solely candidate who makes promises for selfish
of grain products is not adequate for human health. reasons may nonetheless be worthy of the
office for which he or she is running.
22. If the price it pays for coffee beans continues to
increase, the Coffee Shoppe will have to increase its 24. Sociologist: Romantics who claim that people are
prices. In that case, either the Coffee Shoppe will begin not born evil but may be made evil by the imperfect
selling noncoffee products or its coffee sales will institutions that they form cannot be right, for they
decrease. But selling noncoffee products will misunderstand the causal relationship between
decrease the Coffee Shoppe’s overall profitability. people and their institutions. After all, institutions are
merely collections of people. Which one of the
following principles, if valid, would most help to justify
the sociologist’s argument?
(A) People acting together in institutions can do
more good or evil than can people acting individually.
(B) Institutions formed by people are inevitably
imperfect.
(C) People should not be overly optimistic in their
view of individual human beings.
(D) A society’s institutions are the surest gauge of
that society’s values.
(E) The whole does not determine the properties of
the things that compose it.

25. Some anthropologists argue that the human


species could not have survived prehistoric times if
the species had not evolved the ability to cope with
diverse natural environments. However, there is
considerable evidence that Australopithecus
afarensis, a prehistoric species related to early
humans, also thrived in a diverse array of
environments, but became extinct. Hence, the
anthropologists’ claim is false. The reasoning in the
argument is most vulnerable to criticism on the
grounds that the argument
(A) confuses a condition’s being required for a
given result to occur in one case with the condition’s
being sufficient for such a result to occur in a similar
case
(B) takes for granted that if one species had a
characteristic that happened to enable it to
survive certain conditions, at least one related extinct
species must have had the same characteristic
(C) generalizes, from the fact that one species with
a certain characteristic survived certain conditions,
that all related species with the same characteristic
must have survived exactly the same conditions
(D) fails to consider the possibility that
Australopithecus afarensis had one or more
characteristics that lessened its chances of surviving
prehistoric times
(E) fails to consider the possibility that, even if a
condition caused a result to occur in one case, it was
not necessary to cause the result to occur in a similar
case
STOP
IF YOU FINISH BEFORE TIME IS CALLED, YOU MAY
CHECK YOUR WORK ON THIS SECTION ONLY.
DO NOT WORK ON ANY OTHER SECTION IN THE TEST.
SECTION IV prevalent in the U.S., because each genre shares in the
Time—35 minutes nature of
27 Questions (40) the other. Indeed, her poetry offers example after
example of what can only be properly regarded as lyric
Directions: Each set of questions in this section is narrative. Her use of language in these poems is
based on a single passage or a pair of passages. The undeniably lyrical—that is, it evokes emotion and inner
questions are to be answered on the basis of what is states without requiring the reader to organize
stated or implied in the passage or pair of passages. (45) ideas or events in a particular linear structure. Yet
For some of the questions, more than one of the this lyric expression simultaneously presents the
choices could conceivably answer the question. elements of a plot in such a way that the reader is led
However, you are to choose the best answer; that is, repeatedly to take account of clusters of narrative
the response that most accurately and completely details within the lyric flow. Thus while the language
answers the question. is lyrical, it
(50) often comes to constitute, cumulatively, a work of
For decades, there has been a deep rift between poetry narrative fiction. Similarly, many passages in her
and fiction in the United States, especially in academic fiction, though undeniably prose, achieve the status of
settings; graduate writing programs in universities, for lyric narrative through the use of poetic rhythms and
example, train students as poets or as elliptical expression. In short, Dove bridges the gap
(5) writers of fiction, but almost never as both. Both (55) between poetry and fiction not only by writing in
poets and writers of fiction have tended to support this both genres, but also by fusing the two genres within
separation, in large part because the current individual works.
conventional wisdom holds that poetry should be
elliptical and lyrical, reflecting inner states and 1. Which one of the following most accurately
(10) processes of thought or feeling, whereas expresses the main point of the passage?
character and narrative events are the stock-in-trade (A) Rita Dove’s work has been widely acclaimed
of fiction.Certainly it is true that poetry and fiction are primarily because of the lyrical elements she has
distinct genres, but why have specialized education introduced into her fiction.
and literary territoriality resulted from this distinction? (B) Rita Dove’s lyric narratives present clusters of
(15) The answer lies perhaps in a widespread attitude narrative detail in order to create a cumulative
in U.S. culture, which often casts a suspicious eye on narrative without requiring the reader to interpret it in
the generalist. Those with knowledge and expertise in a linear manner.
multiple areas risk charges of dilettantism, as if ability (C) Working against a bias that has long been
in one field is diluted or compromised by dominant in the U.S., recent writers like Rita Dove have
(20) accomplishment in another. Fortunately, there are shown that the lyrical use of language can effectively
signs that the bias against writers who cross generic enhance narrative fiction.
boundaries is diminishing; several recent writers are (D) Unlike many of her U.S. contemporaries, Rita
known and respected for their work in both genres. Dove writes without relying on the traditional
One important example of techniques associated with poetry and fiction.
(25) this trend is Rita Dove, an African American writer (E) Rita Dove’s successful blending of poetry and
highly acclaimed for both her poetry and her fiction. A fiction exemplifies the recent trend away from
few years ago, speaking at a conference entitled the rigid separation of the two genres that has
“Poets Who Write Fiction,” Dove expressed gentle long been prevalent in the U.S.
incredulity about the habit of segregating the genres.
(30) She had grown up reading and loving both fiction 2. Which one of the following is most analogous to the
and poetry, she said, unaware of any purported danger literary achievements that the author attributes to
lurking in attempts to mix the two. She also studied for Dove?
some time in Germany, where, she observes, “Poets (A) A chef combines nontraditional cooking methods
write plays, novelists compose libretti, playwrights and traditional ingredients from
(35) write novels—they would not understand our disparate world cuisines to devise new recipes.
restrictiveness.” It makes little sense, Dove believes, to (B) A professor of film studies becomes a film director
persist in the restrictive approach to poetry and fiction and succeeds, partly due to a wealth
of theoretical knowledge of filmmaking.
(C) An actor who is also a theatrical director teams up 6. In the context of the passage, the author’s primary
with a public health agency to use street theater to purpose in mentioning Dove’s experience in Germany
inform the public about health matters. (lines 32–36) is to
(D) A choreographer defies convention and (A) suggest that the habit of treating poetry and
choreographs dances that combine elements of both fiction as nonoverlapping domains is
ballet and jazz dance. characteristic of English-speaking societies but
(E) A rock musician records several songs from not others
previous decades but introduces extended guitar (B) point to an experience that reinforced Dove’s
solos into each one. conviction that poetry and fiction should not
be rigidly separated
3. According to the passage, in the U.S. there is a (C) indicate that Dove’s strengths as a writer derive
widely held view that in large part from the international character of
(A) poetry should not involve characters or narratives her academic background
(B) unlike the writing of poetry, the writing of fiction is (D) present an illuminating biographical detail about
rarely an academically serious endeavor Dove in an effort to enhance the human
(C) graduate writing programs focus on poetry to interest appeal of the passage
the exclusion of fiction (E) indicate what Dove believes to be the origin of
(D) fiction is most aesthetically effective when it her opposition to the separation of fiction and
incorporates lyrical elements poetry in the U.S.
(E) European literary cultures are suspicious of
generalists 7. It can be inferred from the passage that the author
would be most likely to believe which one of the
4. The author’s attitude toward the deep rift between following?
poetry and fiction in the U.S. can be most accurately (A) Each of Dove’s works can be classified as either
described as one of primarily poetry or primarily fiction, even though it may
(A) perplexity as to what could have led to the contain elements of both.
development of such a rift (B) The aesthetic value of lyric narrative resides in
(B) astonishment that academics have overlooked its representation of a sequence of events, rather than
the existence of the rift in its ability to evoke inner states.
(C) ambivalence toward the effect the rift has had (C) The way in which Dove blends genres in her writing
on U.S. literature is without precedent in U.S. writing.
(D) pessimism regarding the possibility that the rift (D) Narrative that uses lyrical language is generally
can be overcome aesthetically superior to pure lyric poetry.
(E) disapproval of attitudes and presuppositions (E) Writers who successfully cross the generic
underlying the rift boundary between poetry and fiction often try their
hand at genres such as drama as well.
5. In the passage the author conjectures that a cause
of the deep rift between fiction and poetry in the United 8. If this passage had been excerpted from a longer
States may be that text, which one of the following predictions about the
(A) poets and fiction writers each tend to see their near future of U.S. literature would be most likely to
craft as superior to the others’ craft appear in that text?
(B) the methods used in training graduate students (A) The number of writers who write both poetry
in poetry are different from those used in training and fiction will probably continue to grow.
graduate students in other literary fields (B) Because of the increased interest in mixed
(C) publishers often pressure writers to concentrate genres, the small market for pure lyric poetry
on what they do best will likely shrink even further.
(D) a suspicion of generalism deters writers from (C) Narrative poetry will probably come to be
dividing their energies between the two genres regarded as a sub-genre of fiction.
(E) fiction is more widely read and respected than (D) There will probably be a rise in specialization
Poetry among writers in university writing programs.
(E) Writers who continue to work exclusively in the evolutionary origins of human music. Even
poetry or fiction will likely lose their audiences. excluding lullabies, which parents sing to infants,
human mothers and infants under six months
(40) of age engage in ritualized, sequential behaviors,
involving vocal, facial, and bodily interactions. Using
The two passages discuss recent scientific research face-to-face mother-infant interactions filmed at 24
on music. They are adapted from two different papers frames per second, researchers have shown that
presented at a scholarly conference. mothers and infants jointly construct mutually
(45) improvised interactions in which each partner
Passage A tracks the actions of the other. Such episodes last
Did music and human language originate separately from one-half second to three seconds and are
or together? Both systems use intonation and rhythm composed of musical elements—variations in pitch,
to communicate emotions. Both can be produced rhythm, timbre, volume, and tempo.
vocally or with tools, and people can produce (50) What evolutionary advantage would such
(5) both music and language silently to themselves. behavior have? In the course of hominid evolution,
Brain imaging studies suggest that music and brain size increased rapidly. Contemporaneously, the
language are part of one large, vastly complicated, increase in bipedality caused the birth canal to narrow.
neurological system for processing sound. In fact, This resulted in hominid infants being born ever-more
fewer differences than similarities exist between the (55) prematurely, leaving them much more helpless at
(10) neurological processing of the two. One could birth. This helplessness necessitated longer, better
think of the two activities as different radio programs maternal care. Under such conditions, the emotional
that can be broadcast over the same hardware. One bonds created in the premusical mother-infant
noteworthy difference, though, is that, generally interactions we observe in Homo sapiens today—
speaking, people are better at language than music. In behavior whose
music, anyone (60) neurological basis essentially constitutes the
(15) can listen easily enough, but most people do not capacity to make and enjoy music—would have
perform well, and in many cultures composition is left conferred considerable evolutionary advantage.
to specialists. In language, by contrast, nearly
everyone actively performs and composes. Given their 9. Both passages were written primarily in order to
shared neurological basis, it appears answer which one of the following questions?
(20) that music and language evolved together as (A) What evolutionary advantage did larger brain
brain size increased over the course of hominid size confer on early hominids?
evolution. But the primacy of language over music that (B) Why do human mothers and infants engage in
we can observe today suggests that language, not bonding behavior that is composed of musical
music, was the primary function natural selection elements?
operated on. (C) What are the evolutionary origins of the human
(25) Music, it would seem, had little adaptive value of ability to make music?
its own, and most likely developed on the coattails of (D) Do the human abilities to make music and to
language. use language depend on the same neurological
systems?
Passage B (E) Why are most people more adept at using
Darwin claimed that since “neither the enjoyment nor language than they are at making music?
the capacity of producing musical notes are
(30) faculties of the least [practical] use to manthey 10. Each of the two passages mentions the relation of
must be ranked amongst the most mysterious with music to
which he is endowed.” I suggest that the enjoyment of (A) bonding between humans
and the capacity to produce musical notes are (B) human emotion
faculties of indispensable use to mothers and their (C) neurological research
infants and (D) the increasing helplessness of hominid infants
(35) that it is in the emotional bonds created by the (E) the use of tools to produce sounds
interaction of mother and child that we can discover
11. It can be inferred that the authors of the two attempts to answer.
passages would be most likely to disagree over (C) Passage A proposes a hypothesis that passage B
whether attempts to substantiate with new evidence.
(A) the increase in hominid brain size necessitated (D) Passage A expresses a stronger commitment to
earlier births its hypothesis than does passage B.
(B) fewer differences than similarities exist between (E) Passage A and passage B use different evidence
the neurological processing of music and human to support the same conclusion.
language
(C) brain size increased rapidly over the course of
human evolution
(D) the capacity to produce music has great adaptive The World Wide Web, a network of electronically
value to humans produced and interconnected (or “linked”) sites, called
(E) mother-infant bonding involves temporally pages, that are accessible via personal computer,
patterned vocal interactions raises legal issues about the rights of owners of
intellectual
12. The authors would be most likely to agree on the (5) property, notably those who create documents for
answer to which one of the following questions inclusion on Web pages. Some of these owners of
regarding musical capacity in humans? intellectual property claim that unless copyright law is
(A) Does it manifest itself in some form in early strengthened, intellectual property on the Web will not
infancy? be protected from copyright infringement. Web users,
(B) Does it affect the strength of mother-infant bonds? (10) however, claim that if their ability to access
(C) Is it at least partly a result of evolutionary information on Web pages is reduced, the Web cannot
increases in brain size? live up to its potential as an open, interactive medium
(D) Did its evolution spur the development of new of communication. The debate arises from the Web’s
neurological systems? ability to link
(E) Why does it vary so greatly among different (15) one document to another. Links between sites are
individuals? analogous to the inclusion in a printed text of
references to other works, but with one difference: the
13. Which one of the following principles underlies the cited document is instantly retrievable by a user who
arguments in both passages? activates the link. This immediate accessibility
(A) Investigations of the evolutionary origins of human creates
behaviors must take into account the behavior of (20) a problem, since current copyright laws give
nonhuman animals. owners of intellectual property the right to sue a
(B) All human capacities can be explained in terms of distributor of unauthorized copies of their material
the evolutionary advantages they offer. even if that distributor did not personally make the
(C) The fact that a single neurological system copies. If person A, the author of a document, puts the
underlies two different capacities is evidence that document
those capacities evolved concurrently. (25) on a Web page, and person B, the creator of
(D) The discovery of the neurological basis of a human another Web page, creates a link to A’s document, is B
behavior constitutes the discovery of the essence of committing copyright infringement? To answer this
that behavior. question, it must first be determined who controls
(E) The behavior of modern-day humans can provide distribution of a document on
legitimate evidence concerning the evolutionary (30) the Web. When A places a document on a Web
origins of human abilities. page, this is comparable to recording an outgoing
message on one’s telephone answering machine for
14. Which one of the following most accurately others to hear. When B creates a link to A’s document,
characterizes a relationship between the two this is akin to B’s giving out A’s telephone number,
passages? thereby
(A) Passage A and passage B use different evidence (35) allowing third parties to hear the outgoing
to draw divergent conclusions. message for themselves. Anyone who calls can listen
(B) Passage A poses the question that passage B to the message; that is its purpose. While B’s link may
indeed facilitate access to A’s document, the crucial (A) made more restrictive
point is that A, simply by placing that document on the (B) made uniform worldwide
(40) Web, is thereby offering it for distribution. (C) made to impose harsher penalties
Therefore, even if B leads others to the document, it is (D) dutifully enforced
A who actually controls access to it. Hence creating a (E) more fully recognized as legitimate
link to a document is not the same as making or
distributing a copy of that document. Moreover, 17. With which one of the following claims about
techniques are documents placed on Web pages would the author be
(45) already available by which A can restrict access most likely to agree?
to a document. For example, A may require a (A) Such documents cannot receive adequate
password to gain entry to A’s Web page, just as a protection unless current copyright laws are
telephone owner can request an unlisted number and strengthened.
disclose it only to selected parties. Such a solution (B) Such documents cannot be protected from
would compromise unauthorized distribution without significantly
(50) the openness of the Web somewhat, but not as diminishing the potential of the Web to be a widely
much as the threat of copyright infringement used form of communication.
litigation. Changing copyright law to benefit owners of (C) The nearly instantaneous access afforded by the
intellectual property is thus ill-advised because it Web makes it impossible in practice to limit access to
would impede the development of the Web as a public such documents.
(55) forum dedicated to the free exchange of ideas. (D) Such documents can be protected from copyright
infringement with the least damage to the public
15. Which one of the following most accurately interest only by altering existing legal codes.
expresses the main point of the passage? (E) Such documents cannot fully contribute to the
(A) Since distribution of a document placed on a Web’s free exchange of ideas unless their authors
Web page is controlled by the author of that allow them to be freely accessed by those who wish to
page rather than by the person who creates a do so.
link to the page, creating such a link should
not be considered copyright infringement. 18. Based on the passage, the relationship between
(B) Changes in copyright law in response to the strengthening current copyright laws and relying on
development of Web pages and links are passwords to restrict access to a Web document is
ill-advised unless such changes amplify rather most analogous to the relationship between
than restrict the free exchange of ideas (A) allowing everyone use of a public facility and
necessary in a democracy. restricting its use to members of the community
(C) People who are concerned about the access (B) outlawing the use of a drug and outlawing its sale
others may have to the Web documents they (C) prohibiting a sport and relying on participants to
create can easily prevent such access without employ proper safety gear
inhibiting the rights of others to exchange (D) passing a new law and enforcing that law
ideas freely. (E) allowing unrestricted entry to a building and
(D) Problems concerning intellectual property rights restricting entry to those who have been issued a
created by new forms of electronic media are badge
not insuperably difficult to resolve if one applies basic
commonsense principles to these problems. 19. The passage most strongly implies which one of
(E) Maintaining a free exchange of ideas on the Web the following?
offers benefits that far outweigh those that might be (A) There are no creators of links to Web pages who
gained by a small number of individuals if a radical are also owners of intellectual property on Web pages.
alteration of copyright laws aimed at restricting the (B) The person who controls access to a Web page
Web’s growth were allowed. document should be considered the distributor
of that document.
16. Which one of the following is closest in meaning to (C) Rights of privacy should not be extended to
the term “strengthened” as that term is used in line 8 owners of intellectual property placed on the Web.
of the passage? (D) Those who create links to Web pages have
primary control over who reads the documents In tracing the changing face of the Irish landscape,
on those pages. scholars have traditionally relied primarily on evidence
(E) A document on a Web page must be converted to a from historical documents. However, such
physical document via printing before copyright documentary sources provide a fragmentary record at
infringement takes place. (5) best. Reliable accounts are very scarce for many
parts of Ireland prior to the seventeenth century, and
20. According to the passage, which one of the many of the relevant documents from the sixteenth
following features of outgoing messages left on and seventeenth centuries focus selectively on
telephone answering machines is most relevant to the matters relating to military or commercial interests.
debate concerning copyright infringement? (10) Studies of fossilized pollen grains preserved in
(A) Such messages are carried by an electronic peats and lake muds provide an additional means of
medium of communication. investigating vegetative landscape change. Details of
(B) Such messages are not legally protected against changes in vegetation resulting from both human
unauthorized distribution. activities and natural events are reflected in the kinds
(C) Transmission of such messages is virtually (15) and quantities of minute pollen grains that
instantaneous. become trapped in sediments. Analysis of samples
(D) People do not usually care whether or not others can identify which kinds of plants produced the
might record such messages. preserved pollen grains and when they were
(E) Such messages have purposely been made deposited, and in many cases the findings can serve
available to anyone who calls that telephone to supplement or correct
number. (20) the documentary record. For example, analyses of
samples from Long Lough in County Down have
21. The author’s discussion of telephone answering revealed significant patterns of cereal-grain pollen
machines serves primarily to beginning by about 400 A.D. The substantial clay
(A) compare and contrast the legal problems created content of the soil in this part
by two different sorts of electronic media (25) of Down makes cultivation by primitive tools
(B) provide an analogy to illustrate the positions taken difficult. Historians thought that such soils were not
by each of the two sides in the copyright debate tilled to any significant extent until the introduction of
(C) show that the legal problems produced by new the moldboard plough to Ireland in the seventh
communication technology are not themselves new century A.D. Because cereal cultivation would have
(D) illustrate the basic principle the author believes required
should help determine the outcome of the copyright (30) tilling of the soil, the pollen evidence indicates
debate that these soils must indeed have been successfully
(E) show that telephone use also raises concerns tilled before the introduction of the new plough.
about copyright infringement Another example concerns flax cultivation in County
Down, one of the great linen-producing areas
22. According to the passage, present copyright laws (35) of Ireland during the eighteenth century. Some
(A) allow completely unrestricted use of any document aspects of linen production in Down are well
placed by its author on a Web page documented, but the documentary record tells little
(B) allow those who establish links to a document on about the cultivation of flax, the plant from which linen
a Web page to control its distribution to others is made, in that area. The record of eighteenth-century
(C) prohibit anyone but the author of a document from linen
making a profit from the document’s distribution (40) production in Down, together with the knowledge
(D) allow the author of a document to sue anyone who that flax cultivation had been established in Ireland
distributes the document without permission centuries before that time, led some historians to
(E) should be altered to allow more complete freedom surmise that this plant was being cultivated in Down
in the exchange of ideas before the eighteenth century. But pollen analyses
(45) indicate that this is not the case; flax pollen was
found only in deposits laid down since the eighteenth
century. It must be stressed, though, that there are 25. The phrase “documentary record” (lines 20 and 37)
limits to the ability of the pollen record to reflect the primarily refers to
vegetative (A) documented results of analyses of fossilized
(50) history of the landscape. For example, pollen pollen
analyses cannot identify the species, but only the (B) the kinds and quantities of fossilized pollen grains
genus or family, of some plants. Among these is preserved in peats and lake muds
madder, a cultivated dye plant of historical importance (C) written and pictorial descriptions by current
in Ireland. Madder belongs to a plant family that also historians of the events and landscapes of past
comprises centuries
(55) various native weeds, including goosegrass. If (D) government and commercial records, maps, and
madder pollen were present in a deposit it would be similar documents produced in the past that recorded
indistinguishable from that of uncultivated native conditions and events of that time
species. (E) articles, books, and other documents by current
historians listing and analyzing all the available
23. Which one of the following most accurately evidence regarding a particular historical period
expresses the main point of the passage?
(A) Analysis of fossilized pollen is a useful means of 26. The passage indicates that prior to the use of
supplementing and in some cases correcting other pollen analysis in the study of the history of the Irish
sources of information regarding changes landscape, at least some historians believed which
in the Irish landscape. one of the following?
(B) Analyses of historical documents, together with (A) The Irish landscape had experienced significant
pollen evidence, have led to the revision of some flooding during the seventeenth century.
previously accepted hypotheses regarding changes in (B) Cereal grain was not cultivated anywhere in
the Irish landscape. Ireland until at least the seventh century.
(C) Analysis of fossilized pollen has proven to be a (C) The history of the Irish landscape during the
valuable tool in the identification of ancient plant sixteenth and seventeenth centuries was well
species. documented.
(D) Analysis of fossilized pollen has provided new (D) Madder was not used as a dye plant in Ireland
evidence that the cultivation of such crops as cereal until after the eighteenth century.
grains, flax, and madder had a significant impact on (E) The beginning of flax cultivation in County Down
the landscape of Ireland. may well have occurred before the eighteenth century.
(E) While pollen evidence can sometimes supplement
other sources of historical information, its 27. Which one of the following most accurately
applicability is severely limited, since it cannot be used describes the relationship between the second
to identify plant species. paragraph and the final paragraph?
(A) The second paragraph proposes a hypothesis for
24. The passage indicates that pollen analyses have which the final paragraph offers a supporting
provided evidence against which one of the following example.
views? (B) The final paragraph describes a problem that must
(A) The moldboard plough was introduced into Ireland be solved before the method advocated in the second
in the seventh century. paragraph can be considered viable.
(B) In certain parts of County Down, cereal grains (C) The final paragraph qualifies the claim made in the
were not cultivated to any significant extent second paragraph.
before the seventh century. (D) The second paragraph describes a view against
(C) In certain parts of Ireland, cereal grains have been which the author intends to argue, and the final
cultivated continuously since the introduction of the paragraph states the author’s argument against that
moldboard plough. view.
(D) Cereal grain cultivation requires successful (E) The final paragraph offers procedures to
tilling of the soil. supplement the method described in the second
(E) Cereal grain cultivation began in County Down paragraph.
around 400 A.D.
S T O P. IF YOU FINISH BEFORE TIME IS CALLED, YOU (c) 7x-2
MAY CHECK YOUR WORK ON THIS SECTION ONLY. (d) 2-7x
DO NOT WORK ON ANY OTHER SECTION IN THE TEST.
7. Juan is 10 years older than Maria. Ten years ago,
*Although Math was not part of the last Ateneo Law Juan’s age is thrice that of Maria’s. How old will Maria
School Admissions Test, we’ve included this portion be ten years from now?
just in case similar questions appear in this year’s (a) 10
test. (b) 15
MATH PROBLEMS (c) 20
Time—30 minutes (d) 25
25 Questions
8. 72 is 80% of what number?
1. What is 1/2 of 2/3 of 3/5? (a) 75
(a) 1/4 (b) 80
(b) 1/5 (c) 90
(c) 2/15 (d) 95
(d) 6/30
9. A student needs to read 4 chapters of a book that
2. If a case of beer contains 20 to 24 bottles of beer, run from page 184 to page 321. How
what is the least number of beer bottles in five cases many pages does she need to read?
of beer? (a) 137
(a) 90 (b) 138
(b) 100 (c) 147
(c) 110 (d) 148
(d) 120
10. What is the final answer to the following
3. The ₱8.4 billion budget of the University is reduced operations: 9+ (5-3+7)2
by 15%. By how much is the University budget x4 /3 -8?
reduced? (a) 109
(a) ₱1.26 billion (b) 112
(b) ₱2.10 billion (c) 424
(c) ₱7.14 billion (d) 436
(d) ₱9.66 billion
11. Jerry has 150 female cousins and 50 male
4. A dozen of chicos costs ₱16.00. How many chicos cousins. What percentage of his cousins is
can ₱44.00 buy? male?
(a) 30 (a) 10
(b) 32 (b) 15
(c) 33 (c) 20
(d) 36 (d) 25

5. If p/q = 5/7, find the value of q/3p. 12. Monica walks 320 meters in 40 minutes. What
(a) 1 distance can she cover in 7 hours if she
(b) 1/2 walks at the same rate?
(c) 7/5 (a) 3,360
(d) 7/15 (b) 3,260
(c) 3,370
6. A new number is seven less than twice the number. (d) 33,600
If x represents a number, which of these expressions
represents the new number?
(a) 7-2x
(b) 2x-7
13. A bag contains 5 green pens for every 25 red pens. (d) b/a
If the bag contains 250 red pens, how many green
pens are there? 20. What is 40% of 580?
(a) 25 (a) 200
(b) 40 (b) 232
(c) 50 (c) 252
(d) 55 (d) 270

14. 2 cars leave the same place and travel in opposite 21. There are 110 students in a class. If only 11 are
directions. If Car A is traveling at 60 kph and Car B at boys, what percent of the class are girls?
70 kph, in how many minutes will they be 260 km (a) 10%
apart? (b) 11%
(a) 2 (c) 90%
(b) 4 (d) 99%
(c) 60
(d) 120 22. Four carpenters can finish a cabinet in 12 days.
How long will it take for 12 carpenters to finish the
For numbers 15-17: same task?
The scores of ten athletes who joined a competition (a) 3 days
were 80, 82, 83, 85, 89, 89, 89, 90, 93 and 95. (b) 4 days
(c) 4.5 days
15. What is the median? (d) 5 days
(a) 85
(b) 87.5 23. The average grade of 8 students is 89. If one of the
(c) 89 grades is removed, the average grade of the remaining
(d) 90 students is 88. What is the grade that was removed?
(a) 88
16. What is the mode? (b) 89
(a) 85 (c) 90
(b) 87.5 (d) 92
(c) 89
(d) 90 For numbers 24-25:
A textile company manufactures t-shirts and sells
17. What is the mean? them at ₱125 each. The daily overhead cost is ₱66,000
(a) 85 and each shirt costs ₱65 to make.
(b) 87.5
(c) 89 24. If the company makes Y shirts daily, which of these
(d) 90 expressions represents the profit function per day?
(a) 60Y- 66,000
18. Which of the following is divisible by 9? (b) 125Y- 66,000
(a) 1, 839,231 (c) 190Y- 66,000
(b) 2,354, 331 (d) 190Y+ 66,000
(c) 3,456,930
(d) 4,937,492 25. If the company makes 1,500 shirts, how much will
the company profit/lose?
19. If Micka earns A pesos in a year and spends B (a) Profit by P24,000
pesos, what percentage of her annual salary did she (b) Lose by P24,000
save? (c) Profit by P121,000
(a) (a+b)/a (d) Lose by P121,000
(b) (a-b)/b
(c) (a-b)/a
ANSWER KEY 19. A 11. D
20. E 12. C
SECTION I 21. A 13. E
1. A 22. D 14. A
2. C 23. C 15. A
3. C 24. A 16. A
4. E 25. C 17. E
5. E 18. C
6. C SECTION III 19. B
7. A 1. C 20. E
8. D 2. A 21. D
9. E 3. E 22. D
10. D 4. E 23. A
11. A 5. C 24. B
12. A 6. B 25. D
13. D 7. B 26. E
14. E 8. E 27. C
15. A 9. D
16. A 10. E MATH PROBLEMS
17. D 11. E 1. B
18. B 12. D 2. B
19. D 13. B 3. A
20. C 14. C 4. C
21. D 15. C 5. D
22. B 16. B 6. B
23. A 17. B 7. D
18. B 8. C
SECTION II 19. A 9. B
1. B 20. C 10. A
2. B 21. B 11. D
3. D 22. C 12. A
4. A 23. D 13. C
5. B 24. E 14. D
6. B 25. A 15. C
7. D 16. C
8. A SECTION IV 17. B
9. E 1. E 18. A
10. B 2. D 19. C
11. C 3. A 20. B
12. A 4. E 21. C
13. C 5. D 22. B
14. E 6. B 23. D
15. D 7. A 24. A
16. D 8. A 25. A
17. B 9. C
18. B 10. B

Source: Official LSAT Prep Test

You might also like